Cardiology Emergencies Flashcards

1
Q

Which of the following risk factors has been proven to increase the risk of cardiovascular disease?
Question 1 options:

A)

Oral contraceptive use

B)

Type A personality

C)

Stress

D)

Lack of exercise

A

lack of exercise

How well did you know this?
1
Not at all
2
3
4
5
Perfectly
2
Q

Public education about cardiovascular disease focuses on:
Question 2 options:

A)

risk factors and public-access defibrillation.

B)

CPR and signs and symptoms of CVD.

C)

risk factors and signs and symptoms of CVD.

D)

CPR and public access defibrillation.

A

CPR and public access defibrillation

How well did you know this?
1
Not at all
2
3
4
5
Perfectly
3
Q

The valve between the right atrium and right ventricle is the ________ valve.
Question 3 options:

A)

tricuspid

B)

bicuspid

C)

pulmonic

D)

aortic

A

tricuspid

How well did you know this?
1
Not at all
2
3
4
5
Perfectly
4
Q

Blood entering the left atrium arrives via the:
Question 4 options:

A)

bicuspid valve.

B)

pulmonary artery.

C)

superior and inferior vena cava.

D)

pulmonary vein.

A

superior and inferior vena cava

How well did you know this?
1
Not at all
2
3
4
5
Perfectly
5
Q

The first part of the aorta as it leaves the heart is the:
Question 5 options:

A)

thoracic aorta.

B)

ascending aorta.

C)

aortic arch.

D)

descending aorta.

A

ascending aorta

How well did you know this?
1
Not at all
2
3
4
5
Perfectly
6
Q

Your patient is a 54-year-old male who is unresponsive and cyanotic and has agonal respirations. A “quick look” shows ventricular tachycardia. Which of the following is most important when determining the immediate treatment of this patient?
Question 6 options:

A)

Whether he has an implanted cardioverter-defibrillator

B)

How long he has been “down”

C)

Whether he has a pulse

D)

Whether he is allergic to lidocaine

A

whether he has a pulse

How well did you know this?
1
Not at all
2
3
4
5
Perfectly
7
Q

ECG findings associated with hypokalemia include:
Question 7 options:

A)

U waves and an Osborn wave.

B)

an Osborn wave and a “J” wave.

C)

U waves and flat T waves.

D)

flattened T waves and a widened QRS complex.

A

U waves and flat T waves

How well did you know this?
1
Not at all
2
3
4
5
Perfectly
8
Q

The presence of inverted T waves on an ECG indicates:
Question 8 options:

A)

myocardial injury.

B)

myocardial ischemia.

C)

myocardial necrosis.

D)

myocardial infarction.

A

myocardial ischemia

How well did you know this?
1
Not at all
2
3
4
5
Perfectly
9
Q

The pressure in the left ventricle at the end of diastole is called:
Question 9 options:

A)

preload.

B)

ejection fraction.

C)

after.

D)

stroke volume.

A

preload

How well did you know this?
1
Not at all
2
3
4
5
Perfectly
10
Q

Typical stroke volume is about ________ of the volume of the left ventricle.
Question 10 options:

A)

one-third

B)

three-quarters

C)

one-half

D)

two-thirds

A

two-thirds

How well did you know this?
1
Not at all
2
3
4
5
Perfectly
11
Q

Which of the following items is most likely to interfere with the function of an implanted cardiac pacemaker?
Question 11 options:

A)

Magnet

B)

Cellular telephone

C)

Television remote control

D)

Hair dryer

A

magnet

How well did you know this?
1
Not at all
2
3
4
5
Perfectly
12
Q

The first phase of the cardiac cycle is:
Question 12 options:

A)

ejection.

B)

systole.

C)

contraction.

D)

diastole.

A

diastole

How well did you know this?
1
Not at all
2
3
4
5
Perfectly
13
Q

For a resting potential in a cardiac cell to exist, there must be an:
Question 13 options:

A)

ionic equilibrium between the inside and outside of the cell.

B)

influx of calcium ions into the cell.

C)

adequate number of sodium ions inside the cell and potassium ions outside the cell.

D)

adequate number of potassium ions inside the cell and sodium ions outside the cell.

A

adequate number of potassium ions inside the cell and sodium ions outside the cell

How well did you know this?
1
Not at all
2
3
4
5
Perfectly
14
Q

The proportion of the left ventricular volume that is pumped out of the heart during systole is the:
Question 14 options:

A)

ejection fraction.

B)

preload.

C)

stroke volume.

D)

afterload.

A

ejection fraction

How well did you know this?
1
Not at all
2
3
4
5
Perfectly
15
Q

Beta-blockers generally have which of the following effects?
Question 15 options:

A)

Vasoconstriction

B)

Increased cardiac conduction

C)

Decreased myocardial contractility

D)

Increased myocardial contractility

A

decreased myocardial contractility

How well did you know this?
1
Not at all
2
3
4
5
Perfectly
16
Q

Your patient is found sitting on the edge of the bathtub with cool, diaphoretic skin. She states she became lightheaded and nearly “passed out” while vomiting. Your cardiac monitor shows a sinus bradycardia at a rate of 48. Which of the following is most likely?
Question 16 options:

A)

Disease of the cardiac conduction system

B)

Sick sinus syndrome

C)

Use of sympathomimetic medications

D)

Increased parasympathetic tone

A

increased parasympathetic tone

How well did you know this?
1
Not at all
2
3
4
5
Perfectly
17
Q

When the ECG paper is traveling at the standard rate of 25 mm/sec, a large box in the horizontal direction equals:
Question 17 options:

A)

0.20 seconds.

B)

0.12 seconds.

C)

0.24 seconds.

D)

0.08 seconds.

A

0.20 seconds

How well did you know this?
1
Not at all
2
3
4
5
Perfectly
18
Q

Which of the following will occur with an increase in peripheral vascular resistance?
Question 18 options:

A)

Increased preload

B)

Decreased stroke volume

C)

Increased ejection fraction

D)

Decreased afterload

A

decreased stroke volume

How well did you know this?
1
Not at all
2
3
4
5
Perfectly
19
Q

There are 15 small boxes between R waves on an ECG tracing. What is the heart rate?
Question 19 options:

A)

20

B)

120

C)

150

D)

100

A

100

How well did you know this?
1
Not at all
2
3
4
5
Perfectly
20
Q

If the stroke volume decreased, which of the following would occur to maintain the blood pressure at its current value?
Question 20 options:

A)

Increased heart rate and decreased peripheral vascular resistance

B)

Increased heart rate and increased peripheral vascular resistance

C)

Decreased heart rate and decreased peripheral vascular resistance

D)

Decreased heart rate and increased peripheral vascular resistance

A

increased heart rate and increased peripheral vascular resistance

How well did you know this?
1
Not at all
2
3
4
5
Perfectly
21
Q

You have administered a drug with potent beta-1 effects. Which of the following effects should you most anticipate?
Question 21 options:

A)

Increased heart rate

B)

Peripheral vasodilation

C)

Peripheral vasoconstriction

D)

Smooth muscle relaxation

A

increased heart rate

How well did you know this?
1
Not at all
2
3
4
5
Perfectly
22
Q

You have administered a medication to a patient, resulting in a decreased speed of cardiac impulse conduction. This drug is most accurately described as having a ________ effect.
Question 22 options:

A)

negative dromotropic

B)

positive chronotropic

C)

negative chronotropic

D)

positive dromotropic

A

negative dromotropic

How well did you know this?
1
Not at all
2
3
4
5
Perfectly
23
Q

Which of the following endocrine substances acts as a marker for congestive heart failure?
Question 23 options:

A)

Troponin

B)

ACTH

C)

BNP

D)

Angiotensin

A

BNP

How well did you know this?
1
Not at all
2
3
4
5
Perfectly
24
Q

To detect life-threatening cardiac dysrhythmias, the paramedic must view the ECG in ________ lead(s).
Question 24 options:

A)

three

B)

twelve

C)

one

D)

two

A

one

How well did you know this?
1
Not at all
2
3
4
5
Perfectly
25
Q

The total duration of ventricular depolarization is represented by the ________ on the ECG.
Question 25 options:

A)

R-R interval

B)

QRS duration

C)

QT interval

D)

PQ interval

A

QT interval

How well did you know this?
1
Not at all
2
3
4
5
Perfectly
26
Q

Which of the following statements concerning Q waves on the ECG is most accurate?
Question 26 options:

A)

The absence of a Q wave is a significant pathophysiological finding.

B)

A Q wave is only significant in the presence of chest pain.

C)

Q waves are not a normal finding on the ECG.

D)

A Q wave is significant if it is 0.04 or more seconds wide.

A

a Q wave is significant if it is 0.04 or more seconds wide

How well did you know this?
1
Not at all
2
3
4
5
Perfectly
27
Q

Which of the following is most characteristic of right heart failure?
Question 27 options:

A)

JVD, peripheral edema, and liver and spleen engorgement

B)

Ascites, peripheral edema, and cyanosis

C)

JVD, peripheral edema, and pulmonary edema

D)

Rales, cough productive of blood-tinged sputum

A

JVD peripheral edema and liver and spleen engorgement

How well did you know this?
1
Not at all
2
3
4
5
Perfectly
28
Q

The anterior surface of the heart is best viewed by ECG leads:
Question 28 options:

A)

aVR, aVL, and aVF.

B)

I and aVL.

C)

V1-V4.

D)

II, III, and aVF.

A

V1-V4

How well did you know this?
1
Not at all
2
3
4
5
Perfectly
29
Q

While monitoring a patient’s cardiac rhythm, you note that there is no electrical activity after a PQRST complex for a period equal to exactly three of the previous R-R intervals. This is most accurately described as:
Question 29 options:

A)

sinus pause.

B)

sinus block.

C)

sinus arrest.

D)

sinus arrhythmia.

A

sinus block

How well did you know this?
1
Not at all
2
3
4
5
Perfectly
30
Q

A 48-year-old male is sitting upright in bed in respiratory distress. He describes an acute onset of difficulty breathing and chest pain during the night that has been worsening for the past 3 hours. He also complains of nausea. Pain is described as a substernal pressure radiating to his left shoulder. Physical examination reveals cool, diaphoretic skin and rales on auscultation bilaterally. Medical history includes two prior myocardial infarctions. Medications include Zestril and metoprolol. HR = 132, BP = 140/100, RR = 25, SaO2 = 92%. Which of the following is NOT indicated?
Question 30 options:

A)

Nitroglycerin

B)

Adenosine

C)

Morphine

D)

Enalapril

A

Adenosine

How well did you know this?
1
Not at all
2
3
4
5
Perfectly
31
Q

Which of the following is the most likely result of increased pulmonary artery pressure?
Question 31 options:

A)

Increased left ventricular workload and congestive heart failure

B)

Increased left ventricular workload and cor pulmonale

C)

Decreased right atrial workload and right ventricular hypertrophy

D)

Increased right ventricular workload and cor pulmonale

A

increased right ventricular workload and cor pulmonale

How well did you know this?
1
Not at all
2
3
4
5
Perfectly
32
Q

The difference between apical and peripheral pulse rates that results from decreased cardiac output when the atria fail to contract is known as:
Question 32 options:

A)

pulse deficit.

B)

pulsus alternans.

C)

paroxysmal pulse pressure.

D)

pulsus paradoxus.

A

pulse deficit

How well did you know this?
1
Not at all
2
3
4
5
Perfectly
33
Q

Elastic and smooth muscle fibers are primarily found in the tunica ________ of blood vessels.
Question 33 options:

A)

intima

B)

collateralus

C)

media

D)

adventitia

A

media

How well did you know this?
1
Not at all
2
3
4
5
Perfectly
34
Q

________ is a drop in systolic blood pressure of more than 10 mmHg with inspiration.
Question 34 options:

A)

Pulsus paradoxus

B)

Pulse deficit

C)

Pulsus alternans

D)

Electrical alternans

A

pulsus paradoxus

How well did you know this?
1
Not at all
2
3
4
5
Perfectly
35
Q

Which of the following most accurately differentiates cardioversion from defibrillation?
Question 35 options:

A)

Cardioversion cannot be used in patients who have a pulse.

B)

Cardioversion is timed to be synchronous with the patient’s R wave.

C)

Cardioversion requires fewer than 100 joules.

D)

The electrical stimulation of cardioversion travels at a slower rate through the myocardium.

A

cardioversion is timed to be synchronous with the patient’s R wave

How well did you know this?
1
Not at all
2
3
4
5
Perfectly
36
Q

A 67-year-old male is unconscious, is sitting in a chair, and has agonal respirations. His wife states that he was up all night with difficulty breathing and chest discomfort but would not go to the hospital. Physical examination reveals pink, frothy sputum in the airway; cold, diaphoretic skin; and rales audible without a stethoscope. HR = 108, BP = 74 mmHg by palpation, RR = 4, SaO2 = 82%. The monitor shows sinus tachycardia. Which of the following is the highest priority when treating this patient?
Question 36 options:

A)

Intubation

B)

Determining whether the patient has a Do Not Resuscitate order

C)

Dopamine infusion

D)

Immediate transport

A

intubation

How well did you know this?
1
Not at all
2
3
4
5
Perfectly
37
Q

Measures to treat cardiogenic shock include all of the following EXCEPT:
Question 37 options:

A)

reducing peripheral resistance.

B)

increasing the contractile force.

C)

improving preload.

D)

reducing stroke volume.

A

reducing stroke volume

How well did you know this?
1
Not at all
2
3
4
5
Perfectly
38
Q

PSVT is least likely to occur secondary to:
Question 38 options:

A)

Wolff-Parkinson-White syndrome.

B)

myocardial infarction.

C)

stress.

D)

ingestion of caffeine.

A

myocardial infarction

How well did you know this?
1
Not at all
2
3
4
5
Perfectly
39
Q

A 35-year-old male is complaining of a headache, blurred vision, nausea, and vomiting. He has a history of hypertension but is noncompliant with his medications. His pupils are equal and reactive, his skin is warm and dry, and his breath sounds are clear and equal bilaterally. HR = 122, BP = 202/138, RR = 12, SaO2 = 99%. In addition to monitoring his cardiac rhythm, administering oxygen, and starting an IV at a KVO rate, which of the following is most appropriate?
Question 39 options:

A)

Nitroglycerin, SL

B)

Diazepam, IV

C)

Labetalol, IV

D)

Morphine, IV

A

labetalol IV

How well did you know this?
1
Not at all
2
3
4
5
Perfectly
40
Q

Which of the following most accurately describes the etiology of Wolff-Parkinson-White syndrome?
Question 40 options:

A)

Increased automaticity

B)

Idiopathic

C)

Shifting supraventricular pacemaker sites

D)

Presence of an accessory conduction pathway

A

presence of an accessory conduction pathway

How well did you know this?
1
Not at all
2
3
4
5
Perfectly
41
Q

Which of the following is NOT considered a modifiable risk factor for coronary heart disease?
Question 41 options:

A)

Stress

B)

Obesity

C)

Diet

D)

Gender

A

gender

How well did you know this?
1
Not at all
2
3
4
5
Perfectly
42
Q

Signs and symptoms of decreased tissue perfusion secondary to cardiogenic shock include all of the following EXCEPT:
Question 42 options:

A)

tachypnea.

B)

altered mental status.

C)

cold, diaphoretic skin.

D)

constricted pupils.

A

constricted pupils

How well did you know this?
1
Not at all
2
3
4
5
Perfectly
43
Q

Which of the following rhythms requires transcutaneous pacing?
Question 43 options:

A)

Junctional tachycardia

B)

Sinus bradycardia

C)

Supraventricular tachycardia

D)

Symptomatic third-degree AV block

A

symptomatic third degree AV heart block

How well did you know this?
1
Not at all
2
3
4
5
Perfectly
44
Q

A 64-year-old female is alert and oriented, in moderate respiratory distress, and complaining of chest pain. She describes an acute onset of right-sided chest pain that radiates across her chest. Physical examination reveals cold, diaphoretic skin; lung sounds with crackles bilaterally; JVD; and peripheral edema. Medical history includes hypertension, prior myocardial infarction, and heart failure. HR = 128, BP = 86/56, RR = 26, SaO2 = 92%. Which of the following is appropriate in the prehospital treatment of this patient?
Question 44 options:

A)

Dopamine

B)

Nitroglycerin

C)

Morphine

D)

Amiodarone

A

dopamine

How well did you know this?
1
Not at all
2
3
4
5
Perfectly
45
Q

Which of the following is affected by a properly working pacemaker?
Question 45 options:

A)

Cardiac rhythm

B)

Ejection fraction

C)

Stroke volume

D)

Automaticity

A

cardiac rhythm

How well did you know this?
1
Not at all
2
3
4
5
Perfectly
46
Q

An ECG monitor is useful for:
Question 46 options:

A)

determining stroke volume.

B)

evaluating the effectiveness of cardiac contractions.

C)

detecting the total electrical activity within the heart.

D)

determining cardiac output.

A

detecting the total electrical activity within the heart

How well did you know this?
1
Not at all
2
3
4
5
Perfectly
47
Q

The heart sound produced by the closing of the aortic and pulmonary valves is:
Question 47 options:

A)

S2.

B)

S3.

C)

S4.

D)

S1.

A

S2

How well did you know this?
1
Not at all
2
3
4
5
Perfectly
48
Q

The most common cause of death resulting from myocardial infarction is:
Question 48 options:

A)

inadequate tissue perfusion.

B)

end-organ failure.

C)

heart failure.

D)

dysrhythmia.

A

dysrhythmia

How well did you know this?
1
Not at all
2
3
4
5
Perfectly
49
Q

Which of the following most accurately describes a Valsalva maneuver?
Question 49 options:

A)

Immersing the face in cold water

B)

Digital rectal stimulation

C)

Asking the patient to bear down as if to move his bowels with his nose and mouth closed

D)

Firmly pressing the carotid artery against the transverse process of the vertebra behind it

A

asking the patient to bear down as if to move his bowels with his nose and mouth closed

How well did you know this?
1
Not at all
2
3
4
5
Perfectly
50
Q

Which of the following is the least likely cause of PEA?
Question 50 options:

A)

Tension pneumothorax

B)

Cardiac tamponade

C)

Hypovolemia

D)

Hypertension

A

hypertension

How well did you know this?
1
Not at all
2
3
4
5
Perfectly
51
Q

Your patient is a 58-year-old female who is confused and dyspneic. Her daughter called EMS because the patient complained of a fluttering sensation in her chest, followed a few minutes later by chest pain and an acute onset of confusion. She is pale and diaphoretic without a palpable radial pulse. The monitor shows a narrow complex rhythm at a rate of 180. Which of the following is most appropriate?
Question 51 options:

A)

Immediate synchronized cardioversion

B)

Valsalva maneuver

C)

IV adenosine

D)

IV diltiazem

A

immediate synchronized cardioversion

How well did you know this?
1
Not at all
2
3
4
5
Perfectly
52
Q

A junctional escape beat occurs when:
Question 52 options:

A)

the rate of the SA node is slower than that of the AV node.

B)

there is an accessory pathway that causes reentry of the impulse at the AV node.

C)

there is a conduction block between the SA node and AV node.

D)

the AV junction becomes irritable and temporarily overrides the SA node.

A

the rate of the SA nose is slower than that of the AV node

How well did you know this?
1
Not at all
2
3
4
5
Perfectly
53
Q

Release of acetylcholine at the neuroeffector junction would result in a(n):
Question 53 options:

A)

positive dromotropic effect.

B)

negative chronotropic effect.

C)

positive inotropic effect.

D)

increase of both sympathetic and parasympathetic tone.

A

negative chronotropic effect

How well did you know this?
1
Not at all
2
3
4
5
Perfectly
54
Q

Myocardial ischemia may result in:
Question 54 options:

A)

J waves.

B)

ST segment depression.

C)

inverted P waves.

D)

QRS duration greater than 0.12 seconds.

A

inverted P waves

How well did you know this?
1
Not at all
2
3
4
5
Perfectly
55
Q

Auscultation of an S3 is associated with:
Question 55 options:

A)

aortic stenosis.

B)

increased force of atrial contraction.

C)

congestive heart failure.

D)

mitral valve prolapse.

A

congestive heart failure

How well did you know this?
1
Not at all
2
3
4
5
Perfectly
56
Q

Which of the following ECG findings is least anticipated in a patient experiencing an acute myocardial infarction?
Question 56 options:

A)

QRS greater than 0.12 seconds

B)

ST elevation

C)

ST depression

D)

Osborn wave

A

Osborn wave

57
Q

Excessive preload over time would lead to:
Question 57 options:

A)

decreased capacity of the left ventricle.

B)

weakening of the left ventricle.

C)

strengthening of the left ventricle.

D)

increased ejection fraction of the left ventricle.

A

weakening of the left ventricle

58
Q

The base of the heart lies at the level of the ________ rib.
Question 58 options:

A)

third

B)

first

C)

fourth

D)

second

A

second

59
Q

Your patient is a 32-year-old female, 30 weeks’ pregnant, complaining of shortness of breath and palpitations. She is alert and oriented, though anxious; her skin is cool and moist, and her radial pulse is weak and rapid, but regular. HR = 180, BP = 100/72, RR = 24. The monitor shows supraventricular tachycardia. After applying oxygen by nonrebreather and starting an IV of normal saline, which of the following is most appropriate?
Question 59 options:

A)

150 mg of amiodarone

B)

6 mg of adenosine

C)

Transport without further intervention

D)

2 mg of Versed and cardioversion beginning at 50 joules

A

6mg of adenosine

60
Q

Which of the following occurs during depolarization of a cardiac cell?
Question 60 options:

A)

Sodium moves out of the cell.

B)

Potassium moves into the cell.

C)

The cell becomes relatively more positively charged.

D)

The cell becomes negatively charged.

A

the cell becomes relatively more positively charged

61
Q

Poiseuille’s law specifically states that blood flow through a vessel is directly proportional to the ________ of the vessel’s radius.
Question 61 options:

A)

tenth power

B)

third power

C)

square

D)

fourth power

A

fourth power

62
Q

Which of the following is least likely to be associated with the pain of acute myocardial infarction?
Question 62 options:

A)

Radiation to arms and neck

B)

Discomfort lasting longer than 30 minutes

C)

Pain reproducible with palpation

D)

Pain described as sharp

A

pain reducible with palpation

63
Q

A 63-year-old male is alert and oriented, complaining of dizziness. He describes an acute onset of dizziness and near-syncope that has lasted for 15 minutes. He is also experiencing substernal chest pain radiating to his jaw, as well as nausea and weakness. Physical examination reveals cool, diaphoretic skin; delayed capillary refill; and mild crackles to the bases bilaterally. He has no significant medical history, but he takes 325 mg of aspirin a day. HR = 220, BP = 88/52, RR = 16, SaO2 = 92%. Which of the following should be done first?

Question 63 options:

A)

Synchronized cardioversion

B)

Administer 0.4 mg of nitroglycerin sublingually

C)

Start an IV of normal saline

D)

Oxygen by nonrebreathing mask

A

oxygen by nonrebreathing mask

64
Q

Your patient in atrial fibrillation has a heart rate of 108 on the monitor, but her radial pulse is 88. The patient is experiencing:
Question 64 options:

A)

paradoxical pulse.

B)

pulse deficit.

C)

pulsus obliterans.

D)

pulsus alternans.

A

pulse deficit

65
Q

Paramedics use all of the following interventions to treat congestive heart failure EXCEPT:
Question 65 options:

A)

sitting the patient upright.

B)

administering oxygen

C)

decreasing patient anxiety.

D)

administering fluid bolus to treat hypotension.

A

administering fluid bolus to treat hypotension

66
Q

A 48-year-old male is complaining of chest pain that he describes as dull, located substernally, but radiating to his neck. He rates the pain a 6 on a scale of 1 to 10 and complains of nausea and lightheadedness. His skin is cool and diaphoretic. HR = 96, BP = 124/82, RR = 14, SaO2 = 97%. The 12-lead ECG is nondiagnostic. In addition to oxygen, an IV of normal saline at a keep open rate, and transport to the emergency department, which of the following would be most appropriate?
Question 66 options:

A)

Aspirin, nitroglycerin, and morphine

B)

Nitroglycerin, morphine, and furosemide

C)

Nitroglycerin

D)

Reassurance that likely his condition is not cardiac in nature

A

aspirin, nitro, and morphine

67
Q

The amount of resistance that must be overcome by the left ventricle during systole is called:
Question 67 options:

A)

preload.

B)

afterload.

C)

cardiac output.

D)

stroke volume.

A

afterload

68
Q

Signs and/or symptoms of a dissecting thoracic aneurysm include all of the following EXCEPT:
Question 68 options:

A)

palpable pulsating mass.

B)

chest pain.

C)

hypotension.

D)

difficulty breathing.

A

palpable pulsating mass

69
Q

Your patient is a 73-year-old male who is sitting in a recliner, cyanotic, pulseless, and apneic. The patient’s skin is cool and dry. The patient’s wife last saw him an hour and a half ago. Which of the following should you do first?
Question 69 options:

A)

Attach the monitor/defibrillator.

B)

Start CPR.

C)

Check for rigor mortis.

D)

Inform the patient’s wife that he is dead and nothing can be done for him.

A

check for rigor mortis

70
Q

The QRS complex represents:
Question 70 options:

A)

atrial depolarization.

B)

ventricular repolarization.

C)

ventricular depolarization.

D)

atrial repolarization.

A

ventricular depolarization

71
Q

Pharmacological interventions initiated by prehospital care providers in the treatment of congestive heart failure may include all of the following EXCEPT:
Question 71 options:

A)

dopamine.

B)

atropine.

C)

furosemide.

D)

nitroglycerin.

A

atropine

72
Q

Stroke volume × heart rate × systemic vascular resistance =
Question 72 options:

A)

ejection fraction.

B)

cardiac output.

C)

blood pressure.

D)

end-diastolic pressure.

A

blood pressure

73
Q

Cardioversion can be used to treat all of the following rhythms EXCEPT:
Question 73 options:

A)

rapid atrial fibrillation.

B)

supraventricular tachycardia.

C)

ventricular tachycardia with a pulse.

D)

ventricular fibrillation.

A

V fib

74
Q

You have begun transcutaneous pacing of a 52-year-old male who is in third-degree heart block. He was initially unresponsive to all stimuli, with a pulse of 32, blood pressure 60 by palpation, and a respiratory rate of 12. Which of the following is least helpful when determining the effectiveness of transcutaneous pacing?
Question 74 options:

A)

Evidence of mechanical capture

B)

Evidence of electrical capture

C)

Increased blood pressure

D)

Increased level of consciousness

A

evidence of electrical capture

75
Q

Claudication is significant because it:
Question 75 options:

A)

results in inadequate cerebral perfusion.

B)

indicates the presence of atherosclerosis.

C)

results from inadequate coronary artery perfusion.

D)

leads to glaucoma when left untreated.

A

indicates the presence of atherosclerosis

76
Q

Common chief complaints and symptoms associated with cardiac disease include all of the following EXCEPT:
Question 76 options:

A)

chest pain.

B)

syncope.

C)

dyspnea.

D)

vertigo.

A

vertigo

77
Q

The lead to the left of the sternum at the fourth intercostal space is:
Question 77 options:

A)

V3.

B)

V1.

C)

V2.

D)

V4.

A

V2

78
Q

An accelerated junctional rhythm has a rate between ________ and ________.
Question 78 options:

A)

60, 100

B)

100, 150

C)

40, 60

D)

20, 40

A

60, 100

79
Q

Which of the following is most commonly associated with multifocal atrial tachycardia?
Question 79 options:

A)

Pulmonary disease

B)

Use of cocaine, amphetamines, or caffeine

C)

Acute myocardial infarction

D)

Digitalis toxicity

A

pulmonary disease

80
Q

The mitral valve is also known as the ________ valve.
Question 80 options:

A)

right atrioventricular

B)

pulmonic

C)

tricuspid

D)

left atrioventricular

A

left atrioventricular

81
Q

A decrease in preload results in a(n):
Question 81 options:

A)

decrease in peripheral vascular resistance.

B)

decrease in cardiac output.

C)

increase in stroke volume.

D)

decrease in afterload.

A

decrease in cardiac output

82
Q

Your patient has a history of progressively worsening angina that comes on at rest. This most commonly indicates ________ angina.
Question 82 options:

A)

Ludwig’s

B)

Prinzmetal’s

C)

decubitus

D)

unstable angina

A

unstable angina

83
Q

An early sign of hyperkalemia is:
Question 83 options:

A)

widening of the QT interval.

B)

flat T waves on an ECG.

C)

presence of an Osborn wave.

D)

tall, peaked T waves on an ECG.

A

tall, peaked T waves on an ECG

84
Q

The intrinsic firing rate of the AV node is ________ to ________ beats per minute.
Question 84 options:

A)

20, 40

B)

60, 80

C)

80, 100

D)

40, 60

A

40, 60

85
Q

The predominant effect of a drug with primarily alpha properties would result in which of the following?
Question 85 options:

A)

Vasodilation

B)

Increased heart rate

C)

Vasoconstriction

D)

Decreased heart rate

A

vasoconstriction

86
Q
Which of the following is the correct sequence of cardiac electrical activity?
1. AV node
2. Internodal pathways
3. Bundle of His
4. SA node
5. Purkinje fibers
6. Bundle branches
Question 86 options:

A)

4, 2, 1, 3, 6, 5

B)

4, 1, 2, 3, 6, 5

C)

1, 2, 4, 3, 5, 6

D)

1, 2, 4, 3, 6, 5

A

4,1,2,3,6,5

87
Q

A 12-lead ECG that reveals slight ST segment elevation; Q waves in leads II, III, and aVF; and ST elevation in V1 and V2 most indicate which of the following?
Question 87 options:

A)

The patient has had a myocardial infarction in her inferior wall and is experiencing ischemia extending into the right ventricle.

B)

The patient is experiencing myocardial ischemia in her inferior wall that is extending into the septum.

C)

The patient is experiencing ischemia and injury in her left ventricle and septum.

D)

The patient is experiencing myocardial injury in her lateral wall.

A

the patient has had a myocardial infarction in her inferior wall and is experiencing ischemia extending into the right ventricle

88
Q

The pericardial cavity normally holds about ________ mL of ________.
Question 88 options:

A)

1 to 2, serous fluid

B)

30, blood

C)

100, blood

D)

25, straw-colored lubricant

A

25 straw colored lubricant

89
Q

Your patient is a 55-year-old male who is in moderate distress and complaining of chest pain. He states that he experienced an acute onset of left-sided crushing chest pain while playing basketball with his grandson. He complains of weakness and near-syncope with exertion. His skin is pale, cool, and diaphoretic. HR = 40, BP = 72/40, RR = 20, SaO2 = 95%. The monitor shows a third-degree AV block. Which of the following is most appropriate?
Question 89 options:

A)

Atropine, 0.5 mg, up to 2.0 mg

B)

Dopamine at 5 mcg/kg/min

C)

Aspirin, nitroglycerin, and morphine

D)

Sedation and transcutaneous pacing

A

sedation and transcutaneous pacing

90
Q

Increased ________ does NOT occur due to increased venous return to the heart.
Question 90 options:

A)

myocardial contraction

B)

stroke volume

C)

myocardial stretch

D)

afterload

A

afterload

91
Q

Which of the following is NOT a consideration when deciding to withhold resuscitative efforts?
Question 91 options:

A)

Indications of the “down time”

B)

Patient’s age

C)

The nature of injury

D)

Documentation of the patient’s wishes

A

patients age

92
Q

Which of the following ECG findings would indicate a possible pacemaker failure?
Question 92 options:

A)

A pacemaker spike preceding each P wave

B)

Pacemaker spikes without associated QRS complexes

C)

Occasional QRS complexes without pacer spikes

D)

A QRS duration of 0.20 seconds

A

pacemaker spikes without associated QRS complexes

93
Q

An elevation of the ST segment is associated with:
Question 93 options:

A)

left ventricular hypertrophy.

B)

myocardial injury.

C)

slowed conduction through the AV node.

D)

hyperkalemia.

A

myocardial injury

94
Q

The single largest killer of Americans each year is:
Question 94 options:

A)

suicide.

B)

congestive heart failure.

C)

coronary artery disease.

D)

stroke.

A

coronary artery disease

95
Q

Which of the following has NOT been proven to increase the risk of cardiovascular disease?
Question 95 options:

A)

Age

B)

Hypercholesterolemia

C)

Obesity

D)

Smoking

A

obesity

96
Q

The right atrioventricular valve is referred to as the ________ valve.
Question 96 options:

A)

bicuspid

B)

tricuspid

C)

aortic

D)

pulmonary

A

tricuspid

97
Q

The left atrioventricular valve is referred to as the ________ valve.
Question 97 options:

A)

aortic

B)

tricuspid

C)

mitral

D)

pulmonary

A

mitral

98
Q

The right ventricle pushes blood to the lungs through the:
Question 98 options:

A)

pulmonary pathway.

B)

pulmonary vena cava.

C)

pulmonary artery.

D)

pulmonary vein.

A

pulmonary artery

99
Q

During which phase of the cardiac cycle does ventricular filling begin?
Question 99 options:

A)

Refractory

B)

Diastole

C)

Absolute refractory

D)

Systole

A

diastole

100
Q

The pressure in the ventricle at the end of diastole is called:
Question 100 options:

A)

afterload.

B)

stroke volume.

C)

Starling’s law.

D)

preload.

A

preload

101
Q

Starling’s law states that:
Question 101 options:

A)

the greater the contraction, the higher the afterload.

B)

the preload determines the cardiac output.

C)

for every action, there is an equal and opposite reaction.

D)

the more the myocardial muscle is stretched, the greater the contraction.

A

the more the myocardial muscle is stretched, the greater the contraction

102
Q

Cardiac output is:
Question 102 options:

A)

afterload × preload.

B)

stroke volume × diastolic pressure.

C)

stroke volume × heart rate.

D)

heart rate × preload.

A

stroke volume X heart rate

103
Q

Blood pressure is defined as:
Question 103 options:

A)

heart rate × preload × SVR.

B)

afterload × preload × SVE.

C)

stroke volume × heart rate × SVR.

D)

stroke volume × diastolic pressure × SVE.

A

stroke volume X heart rate X SVR

104
Q

Brain natriuretic peptide (BNP) levels are useful in determining:
Question 104 options:

A)

CAD.

B)

COPD.

C)

CVA.

D)

CHF.

A

CHF

105
Q

Cardiac muscle is different from smooth muscle in the fact that it has:
Question 105 options:

A)

peristalsis.

B)

There is no difference.

C)

different nerve pathways.

D)

automaticity.

A

automaticity

106
Q

The intrinsic rate of the SA node is:
Question 106 options:

A)

60-100 bpm.

B)

15-40 bpm.

C)

set by the sympathetic nervous system.

D)

40-60 bpm.

A

60- 100 bpm

107
Q

The intrinsic rate of the AV node is:
Question 107 options:

A)

set by the sympathetic nervous system.

B)

40-60 bpm.

C)

60-100 bpm.

D)

15-40 bpm.

A

40-60 bpm

108
Q

The intrinsic rate of the Purkinje system is:
Question 108 options:

A)

40-60 bpm.

B)

60-100 bpm.

C)

15-40 bpm.

D)

set by the sympathetic nervous system.

A

15-40 bpm

109
Q

On an ECG tracing, positive impulses are seen as:
Question 109 options:

A)

upward deflections.

B)

ST segment.

C)

J point.

D)

downward deflections.

A

upward deflections

110
Q

On an ECG tracing, negative impulses are seen as:
Question 110 options:

A)

upward deflections.

B)

downward deflections.

C)

J point.

D)

ST segment.

A

downward deflections

111
Q

You are evaluating your patient’s ECG under fluorescent lights. You notice significant artifact due to:
Question 111 options:

A)

60 hertz interference.

B)

machine malfunction.

C)

poor eyesight.

D)

road noise.

A

60 hertz interference

112
Q

One small box on the ECG paper indicates:
Question 112 options:

A)

0.20 seconds.

B)

2 seconds.

C)

4 seconds.

D)

0.04 seconds.

A

0.04 seconds

113
Q

One large box on the ECG paper indicates:
Question 113 options:

A)

0.04 seconds.

B)

2 seconds.

C)

0.20 seconds.

D)

4 seconds.

A

0.20 seconds

114
Q

Atrial depolarization is represented on the ECG by the:
Question 114 options:

A)

J point.

B)

ORS.

C)

T wave.

D)

P wave.

A

P waves

115
Q

Ventricular depolarization is represented on the ECG by the:
Question 115 options:

A)

P wave.

B)

QRS complex.

C)

J point.

D)

T wave.

A

QRS complex

116
Q

Ventricular repolarization is represented on the ECG by the:
Question 116 options:

A)

QRS complex.

B)

J point.

C)

T wave.

D)

P wave.

A

T wave

117
Q

Normal interval time for the PR interval is:
Question 117 options:

A)

1.20-2.00 seconds.

B)

0.33-0.42 second.

C)

0.04-0.12 second.

D)

0.12-0.20 second.

A

0.12- 0.20 second

118
Q

Normal interval time for the QRS complex is:
Question 118 options:

A)

0.12-0.20 second.

B)

1.20-2.00 seconds.

C)

0.04-0.12 second.

D)

0.33-0.42 second.

A

0.04- 0.12 seconds

119
Q

Normal interval time for the QT interval is:
Question 119 options:

A)

1.20-2.00 seconds.

B)

0.12-0.20 second.

C)

0.33-0.42 second.

D)

0.04-0.12 second.

A

0.33-0.42 seconds

120
Q

The five-step procedure for analyzing ECGs includes all of the following EXCEPT:
Question 120 options:

A)

QRS complex.

B)

rate.

C)

P wave.

D)

V1.

A

V1

121
Q

Treatment of sinus tachycardia is aimed at:
Question 121 options:

A)

calcium channel blockers.

B)

treating the underlying cause.

C)

adenosine.

D)

beta blockers.

A

treating the underlying cause

122
Q

You are transporting a patient who is having wild changes in his heart rate, an irregular R-R interval, and a history of taking Lanoxin. The patient is now becoming pale, cool, and clammy, with an MAP of 60. You should:
Question 122 options:

A)

not worry, as this is a normal presentation.

B)

start an amiodarone drip.

C)

infuse a calcium channel blocker.

D)

begin TCP and consider a catecholamine infusion.

A

begin TCP consider a catecholamine infusion

123
Q

A potassium level of 3.0 would commonly be associated with which ECG finding?
Question 123 options:

A)

Delta wave

B)

Flattening T wave

C)

T wave merging with QRS

D)

U wave

A

flattening T wave

124
Q

A potassium level of 8.0 would commonly be associated with which ECG finding?
Question 124 options:

A)

Flattening T wave

B)

T wave merging with QRS

C)

U wave

D)

Delta wave

A

t wave merging with QRS

125
Q

Diaphoresis exhibited in a patient actively having an MI is due to:
Question 125 options:

A)

parasympathetic response.

B)

sympathetic response.

C)

increased myocardial demand.

D)

increased metabolism.

A

sympathetic response

126
Q

A parasympatholytic agent used to treat symptomatic bradycardia is:
Question 126 options:

A)

Atrovent.

B)

epinephrine.

C)

Levophed.

D)

atropine.

A

atropine

127
Q

The loading dose of norepinephrine is:
Question 127 options:

A)

2-4 mcg/min.

B)

8-10 mcg/min.

C)

8-10 mg/min.

D)

2-4 mg/ min.

A

8-10 mg/min

128
Q

The loading dose of norepinephrine is:
Question 127 options:

A)

2-4 mcg/min.

B)

8-10 mcg/min.

C)

8-10 mg/min.

D)

2-4 mg/ min.

A

oxygen supply and demand

129
Q

Vasospasms associated with chest pain are known as:
Question 129 options:

A)

variant angina.

B)

Prinzmetal’s angina.

C)

stable angina.

D)

James’ angina.

A

prinzmetals angina

130
Q

A patient with known arteriosclerosis was outside doing lawn work when he started experiencing chest pain. Upon your arrival, the patient states that he has been resting and took a prescribed nitroglycerin tablet 5 minutes ago. The patient states that the pain is subsiding. You suspect:
Question 130 options:

A)

Prinzmetal’s angina.

B)

stable angina.

C)

unstable angina.

D)

myocardial infarction.

A

stable angina

131
Q

Life-threatening arrhythmias can result in death of the AMI patient as soon as:
Question 131 options:

A)

3 hours after onset.

B)

2 hours after onset.

C)

1 hour after onset.

D)

4 hours after onset.

A

1 hour after onset

132
Q

A pathological Q wave indicating infarction should measure:
Question 132 options:

A)

0.04 second.

B)

0.32 second.

C)

0.12 second.

D)

0.004 second.

A

0.04 seconds

133
Q

The maximum window of time a fibrinolytic can be administered is:
Question 133 options:

A)

6 hours after onset of symptoms.

B)

24 hours after onset of symptoms.

C)

12 hours after onset of symptoms.

D)

8 hours after onset of symptoms.

A

6 hours after onset of symptoms

134
Q

Sudden death is described as:
Question 134 options:

A)

any death without a direct known cause.

B)

any death occurring within 1 hour of onset of symptoms.

C)

All death is sudden death.

D)

any death occurring within 3 hours of onset of symptoms.

A

any death occurring within 1 hour of onset of symptoms

135
Q

Which of the following is NOT a phase of cardiac arrest?
Question 135 options:

A)

Electrical phase

B)

Circulatory phase

C)

Rigor mortis phase

D)

Metabolic phase

A

rigor mortis phase

136
Q

You are transporting a cardiac arrest when you achieve ROSC. You now want to keep the patient’s systolic blood pressure in the range of:
Question 136 options:

A)

20-40 mmHg.

B)

120-130 mmHg.

C)

60-70 mmHg.

D)

80-100 mmHg.

A

80-100mmHg

137
Q

You have been on the scene of a 55-year-old cardiac arrest patient for 30 minutes. You have full ALS care initiated, with no response to therapy. You should:
Question 137 options:

A)

keep working until you get a change.

B)

consider defibrillating at 360 joules.

C)

transport immediately.

D)

consider termination of the arrest.

A

consider termination of the arrest

138
Q

You run a 12-lead ECG on a patient exhibiting chest pain. It reveals ST segment elevation in leads II, III, and aVF, with reciprocal in leads aVL and I. You suspect:
Question 138 options:

A)

inferior MI.

B)

lateral MI.

C)

high lateral wall MI.

D)

anterior MI.

A

inferior MI

139
Q

You are called for a patient experiencing SOB. The patient is also a diabetic, so you perform a 12-lead ECG. It shows ST segment elevation in leads V1, V2, V3, and V4 with reciprocal changes in leads V5, V6, II, III, and aVF. You suspect:
Question 139 options:

A)

septal-anterior MI.

B)

inferoseptal MI.

C)

inferior MI.

D)

lateral MI.

A

septal-anterior MI